OPTADS360
AANETWORK
AMBIENT
UREKA
Banner-Video
IN_IMAGE

Tìm giá trị lớn nhất của biểu thức: \(P=\frac{x}{y^2+z^2}-\frac{1}{(x+y+z)^3}\)

Cho x, y, z là các số thực dương thỏa mãn \(5(x^2+y^2+z^2)=9(xy+2yz+zx)\)
Tìm giá trị lớn nhất của biểu thức: \(P=\frac{x}{y^2+z^2}-\frac{1}{(x+y+z)^3}\)

  bởi Bảo Lộc 08/02/2017
ADSENSE/lession_isads=0
QUẢNG CÁO
 

Câu trả lời (1)

  • Từ điều kiện: \(5x^2+ 5(y^2+ z^2) = 9x(y + z) + 18yz\)
                    \(\Leftrightarrow 5x^2- 9x(y + z) = 18yz - 5(y^2+ z^2)\)
    Áp dụng BĐT Côsi ta có: \(yz\leq \frac{1}{4}(y+z)^2;y^2+z^2\geq \frac{1}{2}(y+z)^2\)
    \(\Rightarrow 18yz - 5(y^2+ z^2) \leq 2(y + z)^2\)
    Do đó: \(5x^2- 9x(y + z) \leq 2(y + z)^2 \Leftrightarrow [x - 2(y + z)](5x + y + z) \leq 0\)
    \(\Rightarrow x\leq 2(y+z)\)
    \(P=\frac{x}{y^2+z^2}-\frac{1}{(x+y+z)^3}\leq \frac{2x}{(y+z)^2}-\frac{2}{(x+y+z)^3}\leq \frac{4}{y+z}-\frac{1}{27(y+z)^3}\)
    Đặt \(y + z = t > 0\), ta có: \(P=4t-\frac{1}{27}t^3\)
    Xét hàm \(\Rightarrow P\leq 16\)
    Vậy MaxP = 16 khi \(\left\{\begin{matrix} y=z=\frac{1}{12}\\ x=\frac{1}{3} \end{matrix}\right.\)

      bởi Co Nan 09/02/2017
    Like (0) Báo cáo sai phạm

Nếu bạn hỏi, bạn chỉ thu về một câu trả lời.
Nhưng khi bạn suy nghĩ trả lời, bạn sẽ thu về gấp bội!

Lưu ý: Các trường hợp cố tình spam câu trả lời hoặc bị báo xấu trên 5 lần sẽ bị khóa tài khoản

Gửi câu trả lời Hủy
 
 

Các câu hỏi mới

NONE
OFF